.The measures of the angles of a triangle are in the extended ratio 6: 1: 5. What is the measure of the largest angle?​

Answers

Answer 1

9514 1404 393

Answer:

  90°

Step-by-step explanation:

The largest angle is 6 of the 6+1+5 = 12 ratio units, so is 6/12 = 1/2 of the total number of degrees in a triangle.

The largest angle is 1/2(180°) = 90°.


Related Questions

how many rectangles with an area of 10 square feet can fit within a large rectangle with dimensions 25 feet by 40 feet?​

Answers

Answer:

100 small rectangles

Step-by-step explanation:

Number of small rectangles = Area of large rectangle ÷ area of small rectangle

               [tex]= \frac{25*40}{10}\\\\= 25*4\\= 100[/tex]

need help asap will give brainly

Answers

I believe it is false that is alt int <‘s
THE ANSWER FOR THIS QUESTION IS
True

What word can be used to describe the distribution of this data set: 5, 4, 10, 3, 3, 4, 7, 4, 6, 5, 11, 9, 5, 7. *

Ungrouped data
Tabular data
Central Tendency
Grouped data

Answers

Answer:

Ungrouped data

Step-by-step explanation:

The ungrouped data is the data where it has only individual data points that means it does not have a frequency distribution

So according to the given situation

Since it is mentioned that , 4, 10, 3, 3, 4, 7, 4, 6, 5, 11, 9, 5, 7 so this represent the ungrouped date

The nth term of the sequence is n^2 + 20
How many terms in the sequence are less than 50
plss help!!

Answers

Answer: 5 terms.

Step-by-step explanation:

We know that the n-th term of a sequence is:

Aₙ = n² + 20

Where n can be any natural number, then n ∈ {1, 2, 3, ...}

We want to find how many terms are smaller than 50.

Then we could find the first value of n such that Aₙ is equal or larger than 50.

Then we can solve:

Aₙ = n² + 20 = 50

then:

n² = 50 - 20 = 30

n = √30 = 5.48

This is not a natural number, so we need to round to the next one that is n = 6

This means that the first natural number n such that Aₙ is equal or larger than 50 is n = 6, then for the values:

n = 1, 2, 3, 4, 5

Aₙ is smaller than 50

This means that there are 5 terms of the sequence that are smaller than 50.

can someone help? i rlly need help thank u for anyone that helps ! :)

Answers

Answer:

i think one of them is

(-1,5) and (8,3)

Step-by-step explanation:

just count up how much is on both x and y axes and you'll get ur answer.

goodluck!

also, dont put my answer cuz im afraid u'll get it wrong cuz i tried my best, so jus wait untill someone else answers if u want <3

goodluck

Plz help due tomorrow if correct ill give brailiest

Answers

Division because first you solve whats in the parentheses them you solve and last you divide

Answer:

Division (Hope this helps)

Step-by-step explanation:

circle O has a circumference of 88piecm.
what is the length of the radius of the circle

Answers

Answer:

radius = 9.38 cm   or   7(sq rt 2) cm

Step-by-step explanation:

sq rt 88(Pi) = radius

radius = 9.38 cm   or   7(sq rt 2) cm

Maya runs 8 miles in 84 minutes. At the same rate, how many minutes would she take to run 6 miles?

Answers

Answer:

63 mins

Step-by-step explanation:

84 mins ÷ 8 miles = 10.5 mins per mile

6 miles × 10.5 mins = 63 mins

the answer to your question would be:

please look at the picture below:

I need the answer ASAP

Answers

You can use Photomath

Decide which of the two points lies on the graph of the line.

1. X + Y = 8
A. (2,4) B. (2,6)

2. X = 4
A.(4,2) B. (2,4)

3. Y = X - 2
A. (4,6) B. (6,4)

Answers

Answers:

B) (2,6)A) (4,2)B) (6,4)

===========================================

Explanations:

For this problem, the x and y coordinates must add to 8. In choice A, we have x+y = 2+4 = 6, which doesn't work. But choice B does work since x+y = 2+6 = 8. Therefore, the point (2,6) is located on the line x+y = 8. The equation x+y = 8 is equivalent to y = -x+8.The x coordinate must be equal to 4. The y coordinate can be anything you want. The point (4,2) is on this line, while (2,4) is not. Another point on this line is (4,1). The line is completely vertical.If we plugged the x & y coordinates of choice A into the equation, we get 6 = 4-2 which then becomes 6 = 2 and that's a false statement. That rules out choice A. If we try choice B, then we get 4 = 6-2 which becomes 4 = 4. We get a true statement so choice B is the answer.

Will give brainiest to correct answer with explanation

Suppose 9x + 12 donuts were sold on
Friday and 4x - 3 were sold on Saturday.
What is the total number of donuts sold?

Answers

Answer:

jesus is always the answer

Step-by-step explanation:

jesus jesus

PLEASE HELP! WILL GIVE BRAINLIST!

Jimmy wants to keep track of a friends college savings account and use slope intercept form to build a function, y = mx + b. What information will he need to know to accurately keep track of the account and create a function that will give the number of dollars in the account based on the number of months his friend will save money for college?

A. The initial amount of money use to start the account, and use that deposit for B

B. All of the above

C. The amount of money he will save each month, and use that for slips or m

D. The amount of months he will save money for a college, and use that amount of time for x

Answers

Answer: here is all the answers for assess it #70: day 122

1. F(x) = 225x + 45

2. F (x)= 7x - 105

3. the function would be a curved line if you were to graph it using intercepts or a simple function table

4. The x-intercept is (10,0)

5. All of the above

Are 1/2x+3/4-5/8x-7/8 and 1/8(x+1) equivalent expressions? show your work.

Answers

No, 1/2x+3/4-5/8x-7/8 and 1/8(x+1) are not equivalent expressions.

What is an equivalent expression?

Equivalent expressions are expressions that work the same even though they look different. If two algebraic expressions are equivalent, then the two expressions have the same value when we plug in the same value for the variable.

The given expression is 1/2 x+3/4-5/8 x-7/8

Now, 1/2 x-5/8 x+3/4-7/8

-1/8 x- 1/8

-1/8(x+1)

Therefore, the equivalent expression for the given expression is -1/8(x+1).

To learn more about an equivalent expression visit:

https://brainly.com/question/28170201.

#SPJ1

How many intergers are between √15 and √68​

Answers

Answer:

I think the answer is Four

Step-by-step explanation:

3.9-8= 4,5,6,7

Plzzzzzzz helppp i will give brainliest if right

Answers

Answer:

1.) 20 square inches

2.) 4.16

3.) (-1, -5) (2, -5)

Step-by-step explanation:

13 NEED HELP ASAP WILL GIVE BRAINLY

Answers

Answer:

A. Similar; congruent angles

Step-by-step explanation:

Answer:

The first statement is true because they have congruent angles.

The side length does not matter here because it's useless unless you're trying to find the perimiter of a triangle, which has a completely different algorithm. It is called the Pythagorem Theorum. I just learned the Pythagorem Theorum last class, and it isn't too difficult once you get the hang of it. Wow I went off course. Anyways I hope this helps!

Step-by-step explanation:

Please help.
Algebra.

Answers

9514 1404 393

Answer:

  8

Step-by-step explanation:

A system of linear equations has no solution when the equations describe parallel lines. Parallel lines have the same slope. In your equation, the slope is the x-coefficient, 8. A parallel line will have a slope of 8.

The second equation will have a slope of 8 if the system has no solution.

Rafael drops a ball from the third-story window. This equation represents the approximate height, h, in meters, of the ball above the ground after it falls for t seconds.

h = -5t² + 45

When is the ball at ground level?​

Answers

Answer:

3 seconds is when it will be at ground level.

Step-by-step explanation:

I'll explain it tmr, just reply if you want me to explain.

5-(-2)= helpppp pleaseee

Answers

Answer:

7

Step-by-step explanation:

5-(-2) equals 7

das the answer 7

Someone please help me with this​

Answers

2/3 and 8/12 are proportional but 3/14 and 8/28 aren’t

please help me i need it

Answers

Answer:

-27

Step-by-step explanation:

Draw a big X over the equal sign, then cross multiply the terms to get:

6(p) = 9(p+9)

Then you want to get p by itself.

Multiply the 9 out:6p = 9p+81

Subtract 9p from both sides:-3p = 81

Divide -3 both sides to get p by itself:p = -27

What would x be 3(x-5)=12

Answers

Answer:

x=9

Have a beautiful day!

Hey.... can anybody help me with a question? "If you can change one thing in your community with design, what would it be and how will it work towards racial justice"?

Answer: x=9

Step-by-step explan

PLEASE HELP ME



HAPPY FRIDAY!!!

Answers

Answer:

52.365

Step-by-step explanation:

53.715 + 51.3 + 52.62 = 157.635

210 - 157.635 = 52.365

Nancy’s rectangular garden has a diagonal of 18.4 feet and a width of 7 feet. If she needs to paint the floor, how much area is she going to cover?

Answers

Answer is 17 !! :)))) let me know if you got it right !

a rate


12. A class of 60 is 30% girls. How many girls are there in the class?
b. 180
d. 18
a. 24
c. 240

Answers

Answer:

18

Step-by-step explanation:

30% of 60 is 18

Answer:

24 is the answer

Step-by-step explanation:

you can see in

Which number line shows the solution to the inequality 3x + 6 > 9?
-4-2 0 2 4
-4-2 0 2 4
-4-2 0 2 4
-4-2 0 2 4

Answers

Answer:

its number 3

Step-by-step explanation:

GIVING 20 POINTS, HELP ASAP!! DUE IN 45 MIN!!!!!

Two health clubs offer different pricing plans for their members. Both health clubs charge a one-time sign-up fee and a monthly membership fee. The graph below represents what Health Club B charges.

Answers

The equation of line is solved and Health Club B costs $ 7 less in monthly membership fees that Health Club A

What is an Equation of a line?

The equation of a line is expressed as y = mx + b where m is the slope and b is the y-intercept

And y - y₁ = m ( x - x₁ )

y = y-coordinate of second point

y₁ = y-coordinate of point one

m = slope

x = x-coordinate of second point

x₁ = x-coordinate of point one

The slope m = ( y₂ - y₁ ) / ( x₂ - x₁ )

Given data ,

Let the equation of line be represented as A

Now , the value of A is

Let the first point be P ( 3 , 106 )

Let the second point be Q ( 6 , 187 )

Now , the slope of the line is m = ( y₂ - y₁ ) / ( x₂ - x₁ )

Substituting the values in the equation , we get

Slope m = ( 187 - 106 ) / 3

Slope m = 27

Now , the equation of line is y - y₁ = m ( x - x₁ )

Substituting the values in the equation , we get

y - 106 = 27 ( x - 3 )

y - 106 = 27x - 81

Adding 106 on both sides , we get

y = 27x + 25   be equation (1)

Now , let the equation of line be represented as B

And , the value of B is

Let the first point be R ( 3 , 105 )

Let the second point be S ( 6 , 165 )

Now , the slope of the line is m = ( y₂ - y₁ ) / ( x₂ - x₁ )

Substituting the values in the equation , we get

Slope m = ( 165 - 105 ) / 3

Slope m = 20

Now , the equation of line is y - y₁ = m ( x - x₁ )

Substituting the values in the equation , we get

y - 105 = 20 ( x - 3 )

y - 105 = 20x - 60

Adding 105 on both sides , we get

y = 20x + 45   be equation (2)

Now , when x = 1

Substitute the value of x = 1 in equation (1) and equation (2) , we get

y₁ = 27 + 25 = $ 52

y₂ = 20 + 45 = $ 65

So , health club B costs $ 7 less than health club A every month

Hence , the equation of lines is solved

To learn more about equation of line click :

https://brainly.com/question/14200719

#SPJ1

I NEED HELP WITH THIS!

They plan to sell the small figures for $8 and the large figures for $10. How many of each figure should Pablo and Kenneth create to earn the most amount of money possible? For full credit, explain HOW you got your answer and WHY you think this is the best combination of figures.

Answers

Answer:

Not enough information

Step-by-step explanation:

There is no limit to how many figures you can create. There is not enough information to solve this. If this is only a part of the question, try to include all of it.

Answer: I disagree. If Pablo has enough time to make 10 figures. He could make 8 of the large figures and make $80.00 and then make 2 of the large figures and make $16. Together with the 8 small and 2 large figures they would make $96.00

Step-by-step explanation: I took the test. Dont know if its right, but its better than the other answer.

I need help with this question

Answers

Answer:

In scientific notation is 1.10 7 and expanded 10000000

Can some one please help me with this

Answers

Answer:

I think the answer is B. I hoped I helped a little

Other Questions
What is the mass in grams of 0.250 mol CH2Cl2? URGENTTTWhich of the following best describes how independent assortment results in inherited variations within a species and how it contributes to evolution?A Independent assortment results from the copying of DNA during cell division and causes variations that are sometimes beneficial to a speciesB Independent assortment results from the binary fission of a single cell and causes mutations that can be beneficial or harmful to a speciesC Independent assortment occurs during mitosis and causes uncontrolled cell division which is harmful to a speciesD Independent assortment occurs when chromosomes separate during meiosis and causes variations that can be beneficial or harmful to a species when a grizzly bear hibernates its heart rate drops to 10 beats per minute, which is 20% of its normal value.question: what is a grizzly bear's normal heart rate when not hibernating?__________ beats per minute. Please helpim beging how is the prairie ecosystem similar to the native plant ecosystem? helpppppppppppppppppppppppppppppppp en la oscura habitacion What is the length and direction of your shadow in the morning and afternoon Underlying most of the trade theories discussed is the notion that: Group of answer choices firms that establish a first-mover advantage with regard to the production of a particular new product will dominate global trade in that product. despite a pivotal role in international trade, businesses are typically unable to influence government trade policy. it usually makes sense for a firm to consolidate its productive activities in one country. different countries have particular advantages in different productive activities. At a sale this week, a suit is being sold for $217. This is a 38% discount from the original price. What is the original price? Fascism is an economic and political system where...1. the government lets customers and businesses operate freely.2. the government regulates customers, workers, and businesses for the betterment andinterests of the nation.3. the government owns and operates businesses to ensure fair treatment for theworkers4. the government makes some regulations to ensure businesses are acting fairly andnot harming society Find the slope of the line that passes through (8,7) and (5,5) PLEASE HURRY WILL GIVE BRAINLIEST (30 POINTS)Write a thesis statement for your argumentative essay against online schooli just need one sentence 16 and 3/5 - 4 and 4/5 equals Silvia was growing some plants. She measured the height, in inches, of the plants and made thisline plot to show her data:( please help will give brainliest if you do and do trolling or Ill report you) When should a transfer of receivables be recorded as a sale? The buyer surrenders control of the receivables to the seller. The transferor does not maintain effective control over the transferred assets through an agreement to repurchase or redeem them prior to their maturity. The transferor maintains effective control over the transferred assets through an agreement to repurchase or redeem them prior to their maturity. The transferred assets are isolated from the transferor. The distance around a tree trunk is 69.08 inches. What is the radius of the tree? How many moles of MgO are produced if 0.37 moles of O2 react with excess of Mg? What is E.B.T in writing I need to do a video project for school but dont know what to say. Its asking me for my most memorable event/occurrence at school. For example, Every time I got a perfect score for a test, I got money to buy games. Does anyone have couple of ideas? Anything random is fine. Thank you. (Will mark brainliest and report random answers)